Confusion in Proof of Noether's theoremTransformation of $d^4x$ under translation disregarded?Noether's theorem under arbitrary coordinate transformationNoether's theorem: meaning of transformation of coordinatesNoether's theorem and translationsBig puzzle about Noether's theorem of coordinate transformation (spacetime symmetry)Global and local symmetries in Noether's theorem. And also Stress-Energy tensorsNoether's theorem with infinite parametersVariation of the Action under infinitesimal arbitrary transformations and Noether's TheoremNoether's theorem - Making a global symmetry local (via the SEM tensor)Transformation of coordinates in Noether's TheoremNoether's Theorem in Classical Field theory ConfusionNoether's theorem under arbitrary coordinate transformation

What does the expression "right on the tip of my tongue" mean?

Smallest Guaranteed hash collision cycle length

Why does my circuit work on a breadboard, but not on a perfboard? I am new to soldering

What happens if a creature that would fight isn't on the battlefield anymore?

Why in a Ethernet LAN, a packet sniffer can obtain all packets sent over the LAN?

Two researchers want to work on the same extension to my paper. Who to help?

What is the best way for a skeleton to impersonate human without using magic?

Why did the ICC decide not to probe alleged US atrocities in Afghanistan?

tikz: not so precise graphic

Does Lawful Interception of 4G / the proposed 5G provide a back door for hackers as well?

What are the components of a legend (in the sense of a tale, not a figure legend)?

Why was castling bad for white in this game, and engine strongly prefered trading queens?

How could a Lich maintain the appearance of being alive without magic?

Was this character’s old age look CGI or make-up?

On studying Computer Science vs. Software Engineering to become a proficient coder

Meaning of「〜てみたいと思います」

Plastic-on-plastic lubricant that wont leave a residue?

How does noise-cancellation work in Mac laptops?

How can dragons propel their breath attacks to a long distance

Extracting sublists that contain similar elements

Would an 8% reduction in drag outweigh the weight addition from this custom CFD-tested winglet?

Python Pandas Expand a Column of List of Lists to Two New Column

A cryptic tricolour

Why does getw return -1 when trying to read a character?



Confusion in Proof of Noether's theorem


Transformation of $d^4x$ under translation disregarded?Noether's theorem under arbitrary coordinate transformationNoether's theorem: meaning of transformation of coordinatesNoether's theorem and translationsBig puzzle about Noether's theorem of coordinate transformation (spacetime symmetry)Global and local symmetries in Noether's theorem. And also Stress-Energy tensorsNoether's theorem with infinite parametersVariation of the Action under infinitesimal arbitrary transformations and Noether's TheoremNoether's theorem - Making a global symmetry local (via the SEM tensor)Transformation of coordinates in Noether's TheoremNoether's Theorem in Classical Field theory ConfusionNoether's theorem under arbitrary coordinate transformation













2












$begingroup$


This question is related to this Noether's theorem under arbitrary coordinate transformation and this Transformation of $d^4x$ under translation disregarded?

To proof Noether's theorem every text book that i know assumes that the variation of the action under the trasformation $xmapsto x^prime$is given by
$$delta S=int mathscrL'(phi^prime,partial_muphi^prime)d^4x^prime-int mathscrL(phi,partial_muphi)d^4x$$



For me this variation should always be zero since this is just a change of variables so there is nothing new.
Suppose that in one dimension flat space we have $phi(x)=sin(x)$ and our action is given by
$$S=intsin(x)dx$$
under a trasnlation $xmapsto x+a$ we should have $phi(x) mapsto phi'(x)=sin(x-a)$.



Under a change of coordinates $xmapsto y=x+a$, $phi(x) mapsto phi'(y)=sin(y-a)$ , since $x$ and $y$ are dummy index the action is the same in both case.



Now assuming we have $sqrt g= x^2$. Translating will give us an action
$$S=int x^2sin(x-a)dx$$



But change of variables will give us an action
$$S=int (y-a)^2sin(y-a)dx$$
witch is different.



Shouldn't the variation of the action be written like this $$delta S=int mathscrL'(phi^prime,partial_muphi^prime)d^4x^-int mathscrL(phi,partial_muphi)d^4x$$










share|cite|improve this question











$endgroup$











  • $begingroup$
    What about the Jacobian arising in front of the measure when changing variables?
    $endgroup$
    – Valter Moretti
    May 2 at 7:14










  • $begingroup$
    I don't think I understand the question. By varying the action and setting it to zero one arrives at Euler Lagrange equations. Noether's Theorem is derived by finding the variation in Lagrangian itself. Maybe a little clarification will help.
    $endgroup$
    – Manvendra Somvanshi
    May 2 at 7:16











  • $begingroup$
    Don't we make a transformation of the fields themselves, i.e., $phi(x)tophi(x)+deltaphi(x)$ where the variation $deltaphi(x)$ is an arbitrary variation and, thus, in my understanding, in general, $deltaphi(x)neqfracpartialphi(x)partial x^mudx^mu$. So, it is supposed to be a true change and not just a change of variables, at least, a priori. Correct me if I am mistaken.
    $endgroup$
    – Dvij Mankad
    May 2 at 7:17











  • $begingroup$
    yes that is what i think. but in proof Noether's theorem ,see for example Field Quantization-W. Greiner Eq. 2.38, 2.39, and 2.45. they insist in the variation of the action
    $endgroup$
    – amilton moreira
    May 2 at 7:21











  • $begingroup$
    @Valter Moretti $d^4x^prime=|J|d^4x$
    $endgroup$
    – amilton moreira
    May 2 at 7:28















2












$begingroup$


This question is related to this Noether's theorem under arbitrary coordinate transformation and this Transformation of $d^4x$ under translation disregarded?

To proof Noether's theorem every text book that i know assumes that the variation of the action under the trasformation $xmapsto x^prime$is given by
$$delta S=int mathscrL'(phi^prime,partial_muphi^prime)d^4x^prime-int mathscrL(phi,partial_muphi)d^4x$$



For me this variation should always be zero since this is just a change of variables so there is nothing new.
Suppose that in one dimension flat space we have $phi(x)=sin(x)$ and our action is given by
$$S=intsin(x)dx$$
under a trasnlation $xmapsto x+a$ we should have $phi(x) mapsto phi'(x)=sin(x-a)$.



Under a change of coordinates $xmapsto y=x+a$, $phi(x) mapsto phi'(y)=sin(y-a)$ , since $x$ and $y$ are dummy index the action is the same in both case.



Now assuming we have $sqrt g= x^2$. Translating will give us an action
$$S=int x^2sin(x-a)dx$$



But change of variables will give us an action
$$S=int (y-a)^2sin(y-a)dx$$
witch is different.



Shouldn't the variation of the action be written like this $$delta S=int mathscrL'(phi^prime,partial_muphi^prime)d^4x^-int mathscrL(phi,partial_muphi)d^4x$$










share|cite|improve this question











$endgroup$











  • $begingroup$
    What about the Jacobian arising in front of the measure when changing variables?
    $endgroup$
    – Valter Moretti
    May 2 at 7:14










  • $begingroup$
    I don't think I understand the question. By varying the action and setting it to zero one arrives at Euler Lagrange equations. Noether's Theorem is derived by finding the variation in Lagrangian itself. Maybe a little clarification will help.
    $endgroup$
    – Manvendra Somvanshi
    May 2 at 7:16











  • $begingroup$
    Don't we make a transformation of the fields themselves, i.e., $phi(x)tophi(x)+deltaphi(x)$ where the variation $deltaphi(x)$ is an arbitrary variation and, thus, in my understanding, in general, $deltaphi(x)neqfracpartialphi(x)partial x^mudx^mu$. So, it is supposed to be a true change and not just a change of variables, at least, a priori. Correct me if I am mistaken.
    $endgroup$
    – Dvij Mankad
    May 2 at 7:17











  • $begingroup$
    yes that is what i think. but in proof Noether's theorem ,see for example Field Quantization-W. Greiner Eq. 2.38, 2.39, and 2.45. they insist in the variation of the action
    $endgroup$
    – amilton moreira
    May 2 at 7:21











  • $begingroup$
    @Valter Moretti $d^4x^prime=|J|d^4x$
    $endgroup$
    – amilton moreira
    May 2 at 7:28













2












2








2


1



$begingroup$


This question is related to this Noether's theorem under arbitrary coordinate transformation and this Transformation of $d^4x$ under translation disregarded?

To proof Noether's theorem every text book that i know assumes that the variation of the action under the trasformation $xmapsto x^prime$is given by
$$delta S=int mathscrL'(phi^prime,partial_muphi^prime)d^4x^prime-int mathscrL(phi,partial_muphi)d^4x$$



For me this variation should always be zero since this is just a change of variables so there is nothing new.
Suppose that in one dimension flat space we have $phi(x)=sin(x)$ and our action is given by
$$S=intsin(x)dx$$
under a trasnlation $xmapsto x+a$ we should have $phi(x) mapsto phi'(x)=sin(x-a)$.



Under a change of coordinates $xmapsto y=x+a$, $phi(x) mapsto phi'(y)=sin(y-a)$ , since $x$ and $y$ are dummy index the action is the same in both case.



Now assuming we have $sqrt g= x^2$. Translating will give us an action
$$S=int x^2sin(x-a)dx$$



But change of variables will give us an action
$$S=int (y-a)^2sin(y-a)dx$$
witch is different.



Shouldn't the variation of the action be written like this $$delta S=int mathscrL'(phi^prime,partial_muphi^prime)d^4x^-int mathscrL(phi,partial_muphi)d^4x$$










share|cite|improve this question











$endgroup$




This question is related to this Noether's theorem under arbitrary coordinate transformation and this Transformation of $d^4x$ under translation disregarded?

To proof Noether's theorem every text book that i know assumes that the variation of the action under the trasformation $xmapsto x^prime$is given by
$$delta S=int mathscrL'(phi^prime,partial_muphi^prime)d^4x^prime-int mathscrL(phi,partial_muphi)d^4x$$



For me this variation should always be zero since this is just a change of variables so there is nothing new.
Suppose that in one dimension flat space we have $phi(x)=sin(x)$ and our action is given by
$$S=intsin(x)dx$$
under a trasnlation $xmapsto x+a$ we should have $phi(x) mapsto phi'(x)=sin(x-a)$.



Under a change of coordinates $xmapsto y=x+a$, $phi(x) mapsto phi'(y)=sin(y-a)$ , since $x$ and $y$ are dummy index the action is the same in both case.



Now assuming we have $sqrt g= x^2$. Translating will give us an action
$$S=int x^2sin(x-a)dx$$



But change of variables will give us an action
$$S=int (y-a)^2sin(y-a)dx$$
witch is different.



Shouldn't the variation of the action be written like this $$delta S=int mathscrL'(phi^prime,partial_muphi^prime)d^4x^-int mathscrL(phi,partial_muphi)d^4x$$







symmetry noethers-theorem classical-field-theory






share|cite|improve this question















share|cite|improve this question













share|cite|improve this question




share|cite|improve this question








edited May 2 at 9:10







amilton moreira

















asked May 2 at 6:58









amilton moreiraamilton moreira

377213




377213











  • $begingroup$
    What about the Jacobian arising in front of the measure when changing variables?
    $endgroup$
    – Valter Moretti
    May 2 at 7:14










  • $begingroup$
    I don't think I understand the question. By varying the action and setting it to zero one arrives at Euler Lagrange equations. Noether's Theorem is derived by finding the variation in Lagrangian itself. Maybe a little clarification will help.
    $endgroup$
    – Manvendra Somvanshi
    May 2 at 7:16











  • $begingroup$
    Don't we make a transformation of the fields themselves, i.e., $phi(x)tophi(x)+deltaphi(x)$ where the variation $deltaphi(x)$ is an arbitrary variation and, thus, in my understanding, in general, $deltaphi(x)neqfracpartialphi(x)partial x^mudx^mu$. So, it is supposed to be a true change and not just a change of variables, at least, a priori. Correct me if I am mistaken.
    $endgroup$
    – Dvij Mankad
    May 2 at 7:17











  • $begingroup$
    yes that is what i think. but in proof Noether's theorem ,see for example Field Quantization-W. Greiner Eq. 2.38, 2.39, and 2.45. they insist in the variation of the action
    $endgroup$
    – amilton moreira
    May 2 at 7:21











  • $begingroup$
    @Valter Moretti $d^4x^prime=|J|d^4x$
    $endgroup$
    – amilton moreira
    May 2 at 7:28
















  • $begingroup$
    What about the Jacobian arising in front of the measure when changing variables?
    $endgroup$
    – Valter Moretti
    May 2 at 7:14










  • $begingroup$
    I don't think I understand the question. By varying the action and setting it to zero one arrives at Euler Lagrange equations. Noether's Theorem is derived by finding the variation in Lagrangian itself. Maybe a little clarification will help.
    $endgroup$
    – Manvendra Somvanshi
    May 2 at 7:16











  • $begingroup$
    Don't we make a transformation of the fields themselves, i.e., $phi(x)tophi(x)+deltaphi(x)$ where the variation $deltaphi(x)$ is an arbitrary variation and, thus, in my understanding, in general, $deltaphi(x)neqfracpartialphi(x)partial x^mudx^mu$. So, it is supposed to be a true change and not just a change of variables, at least, a priori. Correct me if I am mistaken.
    $endgroup$
    – Dvij Mankad
    May 2 at 7:17











  • $begingroup$
    yes that is what i think. but in proof Noether's theorem ,see for example Field Quantization-W. Greiner Eq. 2.38, 2.39, and 2.45. they insist in the variation of the action
    $endgroup$
    – amilton moreira
    May 2 at 7:21











  • $begingroup$
    @Valter Moretti $d^4x^prime=|J|d^4x$
    $endgroup$
    – amilton moreira
    May 2 at 7:28















$begingroup$
What about the Jacobian arising in front of the measure when changing variables?
$endgroup$
– Valter Moretti
May 2 at 7:14




$begingroup$
What about the Jacobian arising in front of the measure when changing variables?
$endgroup$
– Valter Moretti
May 2 at 7:14












$begingroup$
I don't think I understand the question. By varying the action and setting it to zero one arrives at Euler Lagrange equations. Noether's Theorem is derived by finding the variation in Lagrangian itself. Maybe a little clarification will help.
$endgroup$
– Manvendra Somvanshi
May 2 at 7:16





$begingroup$
I don't think I understand the question. By varying the action and setting it to zero one arrives at Euler Lagrange equations. Noether's Theorem is derived by finding the variation in Lagrangian itself. Maybe a little clarification will help.
$endgroup$
– Manvendra Somvanshi
May 2 at 7:16













$begingroup$
Don't we make a transformation of the fields themselves, i.e., $phi(x)tophi(x)+deltaphi(x)$ where the variation $deltaphi(x)$ is an arbitrary variation and, thus, in my understanding, in general, $deltaphi(x)neqfracpartialphi(x)partial x^mudx^mu$. So, it is supposed to be a true change and not just a change of variables, at least, a priori. Correct me if I am mistaken.
$endgroup$
– Dvij Mankad
May 2 at 7:17





$begingroup$
Don't we make a transformation of the fields themselves, i.e., $phi(x)tophi(x)+deltaphi(x)$ where the variation $deltaphi(x)$ is an arbitrary variation and, thus, in my understanding, in general, $deltaphi(x)neqfracpartialphi(x)partial x^mudx^mu$. So, it is supposed to be a true change and not just a change of variables, at least, a priori. Correct me if I am mistaken.
$endgroup$
– Dvij Mankad
May 2 at 7:17













$begingroup$
yes that is what i think. but in proof Noether's theorem ,see for example Field Quantization-W. Greiner Eq. 2.38, 2.39, and 2.45. they insist in the variation of the action
$endgroup$
– amilton moreira
May 2 at 7:21





$begingroup$
yes that is what i think. but in proof Noether's theorem ,see for example Field Quantization-W. Greiner Eq. 2.38, 2.39, and 2.45. they insist in the variation of the action
$endgroup$
– amilton moreira
May 2 at 7:21













$begingroup$
@Valter Moretti $d^4x^prime=|J|d^4x$
$endgroup$
– amilton moreira
May 2 at 7:28




$begingroup$
@Valter Moretti $d^4x^prime=|J|d^4x$
$endgroup$
– amilton moreira
May 2 at 7:28










4 Answers
4






active

oldest

votes


















2












$begingroup$

This issue has come up many times on this site. It's one of those things where the standard textbook presentation is severely lacking.



For example, consider translational symmetry. Sometimes this is written as
$$x to x' = x + a, quad phi to phi'(x') = phi(x).$$
However, you don't implement this symmetry transformation by substituting every $x$ with an $x'$ and every $phi$ with a $phi'$, because that's just a trivial change of variables that always leaves the action invariant, much like how a $u$-substitution always leaves an integral invariant. In other words, obviously we have
$$int_-infty^infty x^2 f(x) , dx = int_-infty^infty (x+a)^2 f(x+a) , dx$$
even though $x^2$, which stands in for the Lagrangian here, is not actually translationally invariant.



What they really mean is that we are considering a genuine, nontrivial transformation of the fields. We start with the field $phi$, and end up with the field $phi'$. Indeed in general, we have
$$int_-infty^infty x^2 f(x) , dx neq int_-infty^infty x^2 f(x+a) , dx.$$
You can derive the expression for $phi'$ by thinking "we map the point $x$ to $x'$" but you don't actually do that as well, because then you end up with a trivial transformation. Another way of saying this is that a symmetry means that a system stays the same if you change some things but not other things. In this case, we shifted the function $f$ relative to the fixed background $x^2$ in order to test translational symmetry, which didn’t hold. If you instead changed both, you wouldn't get any useful information.






share|cite|improve this answer











$endgroup$












  • $begingroup$
    you are right. But my question is why the text books insist in this presentation?
    $endgroup$
    – amilton moreira
    May 2 at 8:53











  • $begingroup$
    @amiltonmoreira Because they learned it from other people who also used this bad presentation. It's sad but true: there are about 50 things in the undergraduate physics curriculum that are almost always presented extremely poorly by tradition, and this is one of them. Everybody I know has had to get around each of these potholes themselves.
    $endgroup$
    – knzhou
    May 2 at 8:55










  • $begingroup$
    could you give me a reference where this proof is made right?
    $endgroup$
    – amilton moreira
    May 2 at 8:56










  • $begingroup$
    @amiltonmoreira Sorry, I don't have one from a textbook; if anything they just get more and more sloppy the more advanced they get. But do look at the top answers in the questions you linked earlier, they're good.
    $endgroup$
    – knzhou
    May 2 at 8:59










  • $begingroup$
    @knzhou I think Srednicki's presentation in Chapter $22$ is pretty similar to what you are saying. He doesn't consider any coordinate changes at all and talks purely about true field transformations at each point with the coordinates untouched, i.e., $phi(x)tophi(x)+deltaphi(x)$. This seems to make it manifest that we are not doing some trivial transformations. Correct me if there is some subtle sloppiness I am missing.
    $endgroup$
    – Dvij Mankad
    May 2 at 9:07


















1












$begingroup$

In most physics textbooks (including Noether's own seminal 1918 paper) on Noether's theorem, the infinitesimal variation$^1$
$$ delta S~:=~S_V^prime[phi^prime]- S_V[phi]tagA$$
describes an active (as opposed to passive) infinitesimal transformation $$phi(x)tophi^prime(x^prime), tagB$$ $$ xto x^prime, tagC$$ of the action functional $$ S_V[phi]~=~int_V mathrmd^nx ~cal L(phi(x),partialphi(x),x).tagD$$
The infinitesimal variation (A) needs not vanish in general. However, pure horizontal $x$-variation (C) without vertical $phi$ variation does vanish cf. my Phys.SE answer here.



--



$^1$ The integration region $Vto V^prime$ is conventionally moved according to the horizontal variation (C).






share|cite|improve this answer











$endgroup$




















    0












    $begingroup$

    Noether's theorem states the conservation laws as a function of (the symmetries of) the Lagrangian. The lagrangian is chosen to reflect the symmetries of the physical system it describes. So whether $delta cal L=0$ for a coordinate transformation is a choice.






    share|cite|improve this answer









    $endgroup$




















      0












      $begingroup$


      To proof Noether's theorem every text book that i know assumes that the variation of the action under the trasformation $xmapsto x²$ is given by ...




      I have here only one book where to search for Noether's theorem.
      Itzykson-Zuber do not consider coordinate transformations. They write for space translations (eq. (1-94))
      $$mathscr L(x+a) equiv
      mathscr L(phi_i(x+a), partial_muphi_i(x+a))$$

      where $a$ may also depend on $x$. Variation of fields is written
      $$deltaphi_i = delta a^mu(x),partial_muphi_i(x)$$
      etc. No variation of coordinates in the action integral.



      A final note may help you to clarify the matter. Variational
      formulation in relativistic field theory is but an extension of the
      one in ordinary mechanics. The analogy runs as follows:



      $$matrix
      hfill rm time & rightarrow & rm spacetime coordinates hfillcr
      hfill rm lagrangian coordinates & rightarrow & rm fields. hfill cr$$

      So the relevant variation is the one of fields, not of coordinates.






      share|cite|improve this answer









      $endgroup$













        Your Answer








        StackExchange.ready(function()
        var channelOptions =
        tags: "".split(" "),
        id: "151"
        ;
        initTagRenderer("".split(" "), "".split(" "), channelOptions);

        StackExchange.using("externalEditor", function()
        // Have to fire editor after snippets, if snippets enabled
        if (StackExchange.settings.snippets.snippetsEnabled)
        StackExchange.using("snippets", function()
        createEditor();
        );

        else
        createEditor();

        );

        function createEditor()
        StackExchange.prepareEditor(
        heartbeatType: 'answer',
        autoActivateHeartbeat: false,
        convertImagesToLinks: false,
        noModals: true,
        showLowRepImageUploadWarning: true,
        reputationToPostImages: null,
        bindNavPrevention: true,
        postfix: "",
        imageUploader:
        brandingHtml: "Powered by u003ca class="icon-imgur-white" href="https://imgur.com/"u003eu003c/au003e",
        contentPolicyHtml: "User contributions licensed under u003ca href="https://creativecommons.org/licenses/by-sa/3.0/"u003ecc by-sa 3.0 with attribution requiredu003c/au003e u003ca href="https://stackoverflow.com/legal/content-policy"u003e(content policy)u003c/au003e",
        allowUrls: true
        ,
        noCode: true, onDemand: true,
        discardSelector: ".discard-answer"
        ,immediatelyShowMarkdownHelp:true
        );



        );













        draft saved

        draft discarded


















        StackExchange.ready(
        function ()
        StackExchange.openid.initPostLogin('.new-post-login', 'https%3a%2f%2fphysics.stackexchange.com%2fquestions%2f477328%2fconfusion-in-proof-of-noethers-theorem%23new-answer', 'question_page');

        );

        Post as a guest















        Required, but never shown

























        4 Answers
        4






        active

        oldest

        votes








        4 Answers
        4






        active

        oldest

        votes









        active

        oldest

        votes






        active

        oldest

        votes









        2












        $begingroup$

        This issue has come up many times on this site. It's one of those things where the standard textbook presentation is severely lacking.



        For example, consider translational symmetry. Sometimes this is written as
        $$x to x' = x + a, quad phi to phi'(x') = phi(x).$$
        However, you don't implement this symmetry transformation by substituting every $x$ with an $x'$ and every $phi$ with a $phi'$, because that's just a trivial change of variables that always leaves the action invariant, much like how a $u$-substitution always leaves an integral invariant. In other words, obviously we have
        $$int_-infty^infty x^2 f(x) , dx = int_-infty^infty (x+a)^2 f(x+a) , dx$$
        even though $x^2$, which stands in for the Lagrangian here, is not actually translationally invariant.



        What they really mean is that we are considering a genuine, nontrivial transformation of the fields. We start with the field $phi$, and end up with the field $phi'$. Indeed in general, we have
        $$int_-infty^infty x^2 f(x) , dx neq int_-infty^infty x^2 f(x+a) , dx.$$
        You can derive the expression for $phi'$ by thinking "we map the point $x$ to $x'$" but you don't actually do that as well, because then you end up with a trivial transformation. Another way of saying this is that a symmetry means that a system stays the same if you change some things but not other things. In this case, we shifted the function $f$ relative to the fixed background $x^2$ in order to test translational symmetry, which didn’t hold. If you instead changed both, you wouldn't get any useful information.






        share|cite|improve this answer











        $endgroup$












        • $begingroup$
          you are right. But my question is why the text books insist in this presentation?
          $endgroup$
          – amilton moreira
          May 2 at 8:53











        • $begingroup$
          @amiltonmoreira Because they learned it from other people who also used this bad presentation. It's sad but true: there are about 50 things in the undergraduate physics curriculum that are almost always presented extremely poorly by tradition, and this is one of them. Everybody I know has had to get around each of these potholes themselves.
          $endgroup$
          – knzhou
          May 2 at 8:55










        • $begingroup$
          could you give me a reference where this proof is made right?
          $endgroup$
          – amilton moreira
          May 2 at 8:56










        • $begingroup$
          @amiltonmoreira Sorry, I don't have one from a textbook; if anything they just get more and more sloppy the more advanced they get. But do look at the top answers in the questions you linked earlier, they're good.
          $endgroup$
          – knzhou
          May 2 at 8:59










        • $begingroup$
          @knzhou I think Srednicki's presentation in Chapter $22$ is pretty similar to what you are saying. He doesn't consider any coordinate changes at all and talks purely about true field transformations at each point with the coordinates untouched, i.e., $phi(x)tophi(x)+deltaphi(x)$. This seems to make it manifest that we are not doing some trivial transformations. Correct me if there is some subtle sloppiness I am missing.
          $endgroup$
          – Dvij Mankad
          May 2 at 9:07















        2












        $begingroup$

        This issue has come up many times on this site. It's one of those things where the standard textbook presentation is severely lacking.



        For example, consider translational symmetry. Sometimes this is written as
        $$x to x' = x + a, quad phi to phi'(x') = phi(x).$$
        However, you don't implement this symmetry transformation by substituting every $x$ with an $x'$ and every $phi$ with a $phi'$, because that's just a trivial change of variables that always leaves the action invariant, much like how a $u$-substitution always leaves an integral invariant. In other words, obviously we have
        $$int_-infty^infty x^2 f(x) , dx = int_-infty^infty (x+a)^2 f(x+a) , dx$$
        even though $x^2$, which stands in for the Lagrangian here, is not actually translationally invariant.



        What they really mean is that we are considering a genuine, nontrivial transformation of the fields. We start with the field $phi$, and end up with the field $phi'$. Indeed in general, we have
        $$int_-infty^infty x^2 f(x) , dx neq int_-infty^infty x^2 f(x+a) , dx.$$
        You can derive the expression for $phi'$ by thinking "we map the point $x$ to $x'$" but you don't actually do that as well, because then you end up with a trivial transformation. Another way of saying this is that a symmetry means that a system stays the same if you change some things but not other things. In this case, we shifted the function $f$ relative to the fixed background $x^2$ in order to test translational symmetry, which didn’t hold. If you instead changed both, you wouldn't get any useful information.






        share|cite|improve this answer











        $endgroup$












        • $begingroup$
          you are right. But my question is why the text books insist in this presentation?
          $endgroup$
          – amilton moreira
          May 2 at 8:53











        • $begingroup$
          @amiltonmoreira Because they learned it from other people who also used this bad presentation. It's sad but true: there are about 50 things in the undergraduate physics curriculum that are almost always presented extremely poorly by tradition, and this is one of them. Everybody I know has had to get around each of these potholes themselves.
          $endgroup$
          – knzhou
          May 2 at 8:55










        • $begingroup$
          could you give me a reference where this proof is made right?
          $endgroup$
          – amilton moreira
          May 2 at 8:56










        • $begingroup$
          @amiltonmoreira Sorry, I don't have one from a textbook; if anything they just get more and more sloppy the more advanced they get. But do look at the top answers in the questions you linked earlier, they're good.
          $endgroup$
          – knzhou
          May 2 at 8:59










        • $begingroup$
          @knzhou I think Srednicki's presentation in Chapter $22$ is pretty similar to what you are saying. He doesn't consider any coordinate changes at all and talks purely about true field transformations at each point with the coordinates untouched, i.e., $phi(x)tophi(x)+deltaphi(x)$. This seems to make it manifest that we are not doing some trivial transformations. Correct me if there is some subtle sloppiness I am missing.
          $endgroup$
          – Dvij Mankad
          May 2 at 9:07













        2












        2








        2





        $begingroup$

        This issue has come up many times on this site. It's one of those things where the standard textbook presentation is severely lacking.



        For example, consider translational symmetry. Sometimes this is written as
        $$x to x' = x + a, quad phi to phi'(x') = phi(x).$$
        However, you don't implement this symmetry transformation by substituting every $x$ with an $x'$ and every $phi$ with a $phi'$, because that's just a trivial change of variables that always leaves the action invariant, much like how a $u$-substitution always leaves an integral invariant. In other words, obviously we have
        $$int_-infty^infty x^2 f(x) , dx = int_-infty^infty (x+a)^2 f(x+a) , dx$$
        even though $x^2$, which stands in for the Lagrangian here, is not actually translationally invariant.



        What they really mean is that we are considering a genuine, nontrivial transformation of the fields. We start with the field $phi$, and end up with the field $phi'$. Indeed in general, we have
        $$int_-infty^infty x^2 f(x) , dx neq int_-infty^infty x^2 f(x+a) , dx.$$
        You can derive the expression for $phi'$ by thinking "we map the point $x$ to $x'$" but you don't actually do that as well, because then you end up with a trivial transformation. Another way of saying this is that a symmetry means that a system stays the same if you change some things but not other things. In this case, we shifted the function $f$ relative to the fixed background $x^2$ in order to test translational symmetry, which didn’t hold. If you instead changed both, you wouldn't get any useful information.






        share|cite|improve this answer











        $endgroup$



        This issue has come up many times on this site. It's one of those things where the standard textbook presentation is severely lacking.



        For example, consider translational symmetry. Sometimes this is written as
        $$x to x' = x + a, quad phi to phi'(x') = phi(x).$$
        However, you don't implement this symmetry transformation by substituting every $x$ with an $x'$ and every $phi$ with a $phi'$, because that's just a trivial change of variables that always leaves the action invariant, much like how a $u$-substitution always leaves an integral invariant. In other words, obviously we have
        $$int_-infty^infty x^2 f(x) , dx = int_-infty^infty (x+a)^2 f(x+a) , dx$$
        even though $x^2$, which stands in for the Lagrangian here, is not actually translationally invariant.



        What they really mean is that we are considering a genuine, nontrivial transformation of the fields. We start with the field $phi$, and end up with the field $phi'$. Indeed in general, we have
        $$int_-infty^infty x^2 f(x) , dx neq int_-infty^infty x^2 f(x+a) , dx.$$
        You can derive the expression for $phi'$ by thinking "we map the point $x$ to $x'$" but you don't actually do that as well, because then you end up with a trivial transformation. Another way of saying this is that a symmetry means that a system stays the same if you change some things but not other things. In this case, we shifted the function $f$ relative to the fixed background $x^2$ in order to test translational symmetry, which didn’t hold. If you instead changed both, you wouldn't get any useful information.







        share|cite|improve this answer














        share|cite|improve this answer



        share|cite|improve this answer








        edited May 2 at 15:05

























        answered May 2 at 8:40









        knzhouknzhou

        49.2k12136240




        49.2k12136240











        • $begingroup$
          you are right. But my question is why the text books insist in this presentation?
          $endgroup$
          – amilton moreira
          May 2 at 8:53











        • $begingroup$
          @amiltonmoreira Because they learned it from other people who also used this bad presentation. It's sad but true: there are about 50 things in the undergraduate physics curriculum that are almost always presented extremely poorly by tradition, and this is one of them. Everybody I know has had to get around each of these potholes themselves.
          $endgroup$
          – knzhou
          May 2 at 8:55










        • $begingroup$
          could you give me a reference where this proof is made right?
          $endgroup$
          – amilton moreira
          May 2 at 8:56










        • $begingroup$
          @amiltonmoreira Sorry, I don't have one from a textbook; if anything they just get more and more sloppy the more advanced they get. But do look at the top answers in the questions you linked earlier, they're good.
          $endgroup$
          – knzhou
          May 2 at 8:59










        • $begingroup$
          @knzhou I think Srednicki's presentation in Chapter $22$ is pretty similar to what you are saying. He doesn't consider any coordinate changes at all and talks purely about true field transformations at each point with the coordinates untouched, i.e., $phi(x)tophi(x)+deltaphi(x)$. This seems to make it manifest that we are not doing some trivial transformations. Correct me if there is some subtle sloppiness I am missing.
          $endgroup$
          – Dvij Mankad
          May 2 at 9:07
















        • $begingroup$
          you are right. But my question is why the text books insist in this presentation?
          $endgroup$
          – amilton moreira
          May 2 at 8:53











        • $begingroup$
          @amiltonmoreira Because they learned it from other people who also used this bad presentation. It's sad but true: there are about 50 things in the undergraduate physics curriculum that are almost always presented extremely poorly by tradition, and this is one of them. Everybody I know has had to get around each of these potholes themselves.
          $endgroup$
          – knzhou
          May 2 at 8:55










        • $begingroup$
          could you give me a reference where this proof is made right?
          $endgroup$
          – amilton moreira
          May 2 at 8:56










        • $begingroup$
          @amiltonmoreira Sorry, I don't have one from a textbook; if anything they just get more and more sloppy the more advanced they get. But do look at the top answers in the questions you linked earlier, they're good.
          $endgroup$
          – knzhou
          May 2 at 8:59










        • $begingroup$
          @knzhou I think Srednicki's presentation in Chapter $22$ is pretty similar to what you are saying. He doesn't consider any coordinate changes at all and talks purely about true field transformations at each point with the coordinates untouched, i.e., $phi(x)tophi(x)+deltaphi(x)$. This seems to make it manifest that we are not doing some trivial transformations. Correct me if there is some subtle sloppiness I am missing.
          $endgroup$
          – Dvij Mankad
          May 2 at 9:07















        $begingroup$
        you are right. But my question is why the text books insist in this presentation?
        $endgroup$
        – amilton moreira
        May 2 at 8:53





        $begingroup$
        you are right. But my question is why the text books insist in this presentation?
        $endgroup$
        – amilton moreira
        May 2 at 8:53













        $begingroup$
        @amiltonmoreira Because they learned it from other people who also used this bad presentation. It's sad but true: there are about 50 things in the undergraduate physics curriculum that are almost always presented extremely poorly by tradition, and this is one of them. Everybody I know has had to get around each of these potholes themselves.
        $endgroup$
        – knzhou
        May 2 at 8:55




        $begingroup$
        @amiltonmoreira Because they learned it from other people who also used this bad presentation. It's sad but true: there are about 50 things in the undergraduate physics curriculum that are almost always presented extremely poorly by tradition, and this is one of them. Everybody I know has had to get around each of these potholes themselves.
        $endgroup$
        – knzhou
        May 2 at 8:55












        $begingroup$
        could you give me a reference where this proof is made right?
        $endgroup$
        – amilton moreira
        May 2 at 8:56




        $begingroup$
        could you give me a reference where this proof is made right?
        $endgroup$
        – amilton moreira
        May 2 at 8:56












        $begingroup$
        @amiltonmoreira Sorry, I don't have one from a textbook; if anything they just get more and more sloppy the more advanced they get. But do look at the top answers in the questions you linked earlier, they're good.
        $endgroup$
        – knzhou
        May 2 at 8:59




        $begingroup$
        @amiltonmoreira Sorry, I don't have one from a textbook; if anything they just get more and more sloppy the more advanced they get. But do look at the top answers in the questions you linked earlier, they're good.
        $endgroup$
        – knzhou
        May 2 at 8:59












        $begingroup$
        @knzhou I think Srednicki's presentation in Chapter $22$ is pretty similar to what you are saying. He doesn't consider any coordinate changes at all and talks purely about true field transformations at each point with the coordinates untouched, i.e., $phi(x)tophi(x)+deltaphi(x)$. This seems to make it manifest that we are not doing some trivial transformations. Correct me if there is some subtle sloppiness I am missing.
        $endgroup$
        – Dvij Mankad
        May 2 at 9:07




        $begingroup$
        @knzhou I think Srednicki's presentation in Chapter $22$ is pretty similar to what you are saying. He doesn't consider any coordinate changes at all and talks purely about true field transformations at each point with the coordinates untouched, i.e., $phi(x)tophi(x)+deltaphi(x)$. This seems to make it manifest that we are not doing some trivial transformations. Correct me if there is some subtle sloppiness I am missing.
        $endgroup$
        – Dvij Mankad
        May 2 at 9:07











        1












        $begingroup$

        In most physics textbooks (including Noether's own seminal 1918 paper) on Noether's theorem, the infinitesimal variation$^1$
        $$ delta S~:=~S_V^prime[phi^prime]- S_V[phi]tagA$$
        describes an active (as opposed to passive) infinitesimal transformation $$phi(x)tophi^prime(x^prime), tagB$$ $$ xto x^prime, tagC$$ of the action functional $$ S_V[phi]~=~int_V mathrmd^nx ~cal L(phi(x),partialphi(x),x).tagD$$
        The infinitesimal variation (A) needs not vanish in general. However, pure horizontal $x$-variation (C) without vertical $phi$ variation does vanish cf. my Phys.SE answer here.



        --



        $^1$ The integration region $Vto V^prime$ is conventionally moved according to the horizontal variation (C).






        share|cite|improve this answer











        $endgroup$

















          1












          $begingroup$

          In most physics textbooks (including Noether's own seminal 1918 paper) on Noether's theorem, the infinitesimal variation$^1$
          $$ delta S~:=~S_V^prime[phi^prime]- S_V[phi]tagA$$
          describes an active (as opposed to passive) infinitesimal transformation $$phi(x)tophi^prime(x^prime), tagB$$ $$ xto x^prime, tagC$$ of the action functional $$ S_V[phi]~=~int_V mathrmd^nx ~cal L(phi(x),partialphi(x),x).tagD$$
          The infinitesimal variation (A) needs not vanish in general. However, pure horizontal $x$-variation (C) without vertical $phi$ variation does vanish cf. my Phys.SE answer here.



          --



          $^1$ The integration region $Vto V^prime$ is conventionally moved according to the horizontal variation (C).






          share|cite|improve this answer











          $endgroup$















            1












            1








            1





            $begingroup$

            In most physics textbooks (including Noether's own seminal 1918 paper) on Noether's theorem, the infinitesimal variation$^1$
            $$ delta S~:=~S_V^prime[phi^prime]- S_V[phi]tagA$$
            describes an active (as opposed to passive) infinitesimal transformation $$phi(x)tophi^prime(x^prime), tagB$$ $$ xto x^prime, tagC$$ of the action functional $$ S_V[phi]~=~int_V mathrmd^nx ~cal L(phi(x),partialphi(x),x).tagD$$
            The infinitesimal variation (A) needs not vanish in general. However, pure horizontal $x$-variation (C) without vertical $phi$ variation does vanish cf. my Phys.SE answer here.



            --



            $^1$ The integration region $Vto V^prime$ is conventionally moved according to the horizontal variation (C).






            share|cite|improve this answer











            $endgroup$



            In most physics textbooks (including Noether's own seminal 1918 paper) on Noether's theorem, the infinitesimal variation$^1$
            $$ delta S~:=~S_V^prime[phi^prime]- S_V[phi]tagA$$
            describes an active (as opposed to passive) infinitesimal transformation $$phi(x)tophi^prime(x^prime), tagB$$ $$ xto x^prime, tagC$$ of the action functional $$ S_V[phi]~=~int_V mathrmd^nx ~cal L(phi(x),partialphi(x),x).tagD$$
            The infinitesimal variation (A) needs not vanish in general. However, pure horizontal $x$-variation (C) without vertical $phi$ variation does vanish cf. my Phys.SE answer here.



            --



            $^1$ The integration region $Vto V^prime$ is conventionally moved according to the horizontal variation (C).







            share|cite|improve this answer














            share|cite|improve this answer



            share|cite|improve this answer








            edited May 2 at 9:07

























            answered May 2 at 8:24









            QmechanicQmechanic

            109k122041262




            109k122041262





















                0












                $begingroup$

                Noether's theorem states the conservation laws as a function of (the symmetries of) the Lagrangian. The lagrangian is chosen to reflect the symmetries of the physical system it describes. So whether $delta cal L=0$ for a coordinate transformation is a choice.






                share|cite|improve this answer









                $endgroup$

















                  0












                  $begingroup$

                  Noether's theorem states the conservation laws as a function of (the symmetries of) the Lagrangian. The lagrangian is chosen to reflect the symmetries of the physical system it describes. So whether $delta cal L=0$ for a coordinate transformation is a choice.






                  share|cite|improve this answer









                  $endgroup$















                    0












                    0








                    0





                    $begingroup$

                    Noether's theorem states the conservation laws as a function of (the symmetries of) the Lagrangian. The lagrangian is chosen to reflect the symmetries of the physical system it describes. So whether $delta cal L=0$ for a coordinate transformation is a choice.






                    share|cite|improve this answer









                    $endgroup$



                    Noether's theorem states the conservation laws as a function of (the symmetries of) the Lagrangian. The lagrangian is chosen to reflect the symmetries of the physical system it describes. So whether $delta cal L=0$ for a coordinate transformation is a choice.







                    share|cite|improve this answer












                    share|cite|improve this answer



                    share|cite|improve this answer










                    answered May 2 at 8:26









                    my2ctsmy2cts

                    6,3542722




                    6,3542722





















                        0












                        $begingroup$


                        To proof Noether's theorem every text book that i know assumes that the variation of the action under the trasformation $xmapsto x²$ is given by ...




                        I have here only one book where to search for Noether's theorem.
                        Itzykson-Zuber do not consider coordinate transformations. They write for space translations (eq. (1-94))
                        $$mathscr L(x+a) equiv
                        mathscr L(phi_i(x+a), partial_muphi_i(x+a))$$

                        where $a$ may also depend on $x$. Variation of fields is written
                        $$deltaphi_i = delta a^mu(x),partial_muphi_i(x)$$
                        etc. No variation of coordinates in the action integral.



                        A final note may help you to clarify the matter. Variational
                        formulation in relativistic field theory is but an extension of the
                        one in ordinary mechanics. The analogy runs as follows:



                        $$matrix
                        hfill rm time & rightarrow & rm spacetime coordinates hfillcr
                        hfill rm lagrangian coordinates & rightarrow & rm fields. hfill cr$$

                        So the relevant variation is the one of fields, not of coordinates.






                        share|cite|improve this answer









                        $endgroup$

















                          0












                          $begingroup$


                          To proof Noether's theorem every text book that i know assumes that the variation of the action under the trasformation $xmapsto x²$ is given by ...




                          I have here only one book where to search for Noether's theorem.
                          Itzykson-Zuber do not consider coordinate transformations. They write for space translations (eq. (1-94))
                          $$mathscr L(x+a) equiv
                          mathscr L(phi_i(x+a), partial_muphi_i(x+a))$$

                          where $a$ may also depend on $x$. Variation of fields is written
                          $$deltaphi_i = delta a^mu(x),partial_muphi_i(x)$$
                          etc. No variation of coordinates in the action integral.



                          A final note may help you to clarify the matter. Variational
                          formulation in relativistic field theory is but an extension of the
                          one in ordinary mechanics. The analogy runs as follows:



                          $$matrix
                          hfill rm time & rightarrow & rm spacetime coordinates hfillcr
                          hfill rm lagrangian coordinates & rightarrow & rm fields. hfill cr$$

                          So the relevant variation is the one of fields, not of coordinates.






                          share|cite|improve this answer









                          $endgroup$















                            0












                            0








                            0





                            $begingroup$


                            To proof Noether's theorem every text book that i know assumes that the variation of the action under the trasformation $xmapsto x²$ is given by ...




                            I have here only one book where to search for Noether's theorem.
                            Itzykson-Zuber do not consider coordinate transformations. They write for space translations (eq. (1-94))
                            $$mathscr L(x+a) equiv
                            mathscr L(phi_i(x+a), partial_muphi_i(x+a))$$

                            where $a$ may also depend on $x$. Variation of fields is written
                            $$deltaphi_i = delta a^mu(x),partial_muphi_i(x)$$
                            etc. No variation of coordinates in the action integral.



                            A final note may help you to clarify the matter. Variational
                            formulation in relativistic field theory is but an extension of the
                            one in ordinary mechanics. The analogy runs as follows:



                            $$matrix
                            hfill rm time & rightarrow & rm spacetime coordinates hfillcr
                            hfill rm lagrangian coordinates & rightarrow & rm fields. hfill cr$$

                            So the relevant variation is the one of fields, not of coordinates.






                            share|cite|improve this answer









                            $endgroup$




                            To proof Noether's theorem every text book that i know assumes that the variation of the action under the trasformation $xmapsto x²$ is given by ...




                            I have here only one book where to search for Noether's theorem.
                            Itzykson-Zuber do not consider coordinate transformations. They write for space translations (eq. (1-94))
                            $$mathscr L(x+a) equiv
                            mathscr L(phi_i(x+a), partial_muphi_i(x+a))$$

                            where $a$ may also depend on $x$. Variation of fields is written
                            $$deltaphi_i = delta a^mu(x),partial_muphi_i(x)$$
                            etc. No variation of coordinates in the action integral.



                            A final note may help you to clarify the matter. Variational
                            formulation in relativistic field theory is but an extension of the
                            one in ordinary mechanics. The analogy runs as follows:



                            $$matrix
                            hfill rm time & rightarrow & rm spacetime coordinates hfillcr
                            hfill rm lagrangian coordinates & rightarrow & rm fields. hfill cr$$

                            So the relevant variation is the one of fields, not of coordinates.







                            share|cite|improve this answer












                            share|cite|improve this answer



                            share|cite|improve this answer










                            answered May 2 at 14:05









                            Elio FabriElio Fabri

                            4,0671214




                            4,0671214



























                                draft saved

                                draft discarded
















































                                Thanks for contributing an answer to Physics Stack Exchange!


                                • Please be sure to answer the question. Provide details and share your research!

                                But avoid


                                • Asking for help, clarification, or responding to other answers.

                                • Making statements based on opinion; back them up with references or personal experience.

                                Use MathJax to format equations. MathJax reference.


                                To learn more, see our tips on writing great answers.




                                draft saved


                                draft discarded














                                StackExchange.ready(
                                function ()
                                StackExchange.openid.initPostLogin('.new-post-login', 'https%3a%2f%2fphysics.stackexchange.com%2fquestions%2f477328%2fconfusion-in-proof-of-noethers-theorem%23new-answer', 'question_page');

                                );

                                Post as a guest















                                Required, but never shown





















































                                Required, but never shown














                                Required, but never shown












                                Required, but never shown







                                Required, but never shown

































                                Required, but never shown














                                Required, but never shown












                                Required, but never shown







                                Required, but never shown







                                Popular posts from this blog

                                Wikipedia:Vital articles Мазмуну Biography - Өмүр баян Philosophy and psychology - Философия жана психология Religion - Дин Social sciences - Коомдук илимдер Language and literature - Тил жана адабият Science - Илим Technology - Технология Arts and recreation - Искусство жана эс алуу History and geography - Тарых жана география Навигация менюсу

                                Bruxelas-Capital Índice Historia | Composición | Situación lingüística | Clima | Cidades irmandadas | Notas | Véxase tamén | Menú de navegacióneO uso das linguas en Bruxelas e a situación do neerlandés"Rexión de Bruxelas Capital"o orixinalSitio da rexiónPáxina de Bruselas no sitio da Oficina de Promoción Turística de Valonia e BruxelasMapa Interactivo da Rexión de Bruxelas-CapitaleeWorldCat332144929079854441105155190212ID28008674080552-90000 0001 0666 3698n94104302ID540940339365017018237

                                What should I write in an apology letter, since I have decided not to join a company after accepting an offer letterShould I keep looking after accepting a job offer?What should I do when I've been verbally told I would get an offer letter, but still haven't gotten one after 4 weeks?Do I accept an offer from a company that I am not likely to join?New job hasn't confirmed starting date and I want to give current employer as much notice as possibleHow should I address my manager in my resignation letter?HR delayed background verification, now jobless as resignedNo email communication after accepting a formal written offer. How should I phrase the call?What should I do if after receiving a verbal offer letter I am informed that my written job offer is put on hold due to some internal issues?Should I inform the current employer that I am about to resign within 1-2 weeks since I have signed the offer letter and waiting for visa?What company will do, if I send their offer letter to another company